LSAT and Law School Admissions Forum

Get expert LSAT preparation and law school admissions advice from PowerScore Test Preparation.

 Administrator
PowerScore Staff
  • PowerScore Staff
  • Posts: 8917
  • Joined: Feb 02, 2011
|
#32011
Complete Question Explanation
(The complete setup for this game can be found here: lsat/viewtopic.php?t=15241)

The correct answer choice is (A)

Similar to question #10, if J is assigned to area 2, first consider what would happen if O is in 2 with it: that would force J and K together (closing area 2), which moves L to area 3 with M, and leaves P alone in area 1. But this violates the second rule, so we cannot have O and J together in area 2 (this is seen in Template 2):
PT79_Game_#2_#12_diagram 1.png
Next, consider the implications of the last rule. Since O is not assigned to area 2, we need to ensure that J is assigned to a different area than K. Therefore, K is not assigned to area 2. Furthermore, K cannot be in area 3 (ever, as we’ve discussed, and certainly not here), as then L would not be with either K or M. Consequently, K must be assigned to area 1, proving answer choice (A):
PT79_Game_#2_#12_diagram 2.png
Answer choice (B) is incorrect, because L can also be assigned to area 3.

Answer choice (C) is incorrect, because O must be assigned to area 3.

Answer choice (D): This answer choice is incorrect, because P can also be assigned to area 3.

Answer choice (E): This answer choice is incorrect, because P can also be assigned to area 2.

Get the most out of your LSAT Prep Plus subscription.

Analyze and track your performance with our Testing and Analytics Package.